2003 AIME I Problems/Problem 12
Problem
In convex quadrilateral
and
The perimeter of
is
. Find
(The notation
means the greatest integer that is less than or equal to
)
Solution
Solution 1
![[asy] real x = 1.60; /* arbitrary */ pointpen = black; pathpen = black+linewidth(0.7); size(180); real BD = x*x + 1.80*1.80 - 2 * 1.80 * x * 7 / 9; pair A=(0,0),B=(1.8,0),D=IP(CR(A,x),CR(B,BD)),C=OP(CR(D,1.8),CR(B,2.80 - x)); D(MP("A",A)--MP("B",B)--MP("C",C)--MP("D",D,N)--B--A--D); MP("180",(A+B)/2); MP("180",(C+D)/2,NE); D(anglemark(B,A,D)); D(anglemark(D,C,B)); [/asy]](http://latex.artofproblemsolving.com/0/6/0/060d4d918b4f89b6f156f9af44aea8da1238fcdd.png)
By the Law of Cosines on
at angle
and on
at angle
(note
),
We know that
.
.
Solution 2
Notice that
, and
, and
, so we have side-side-angle matching on triangles
and
. Since the problem does not allow
, we know that
is not a right angle, and there is a unique other triangle with the matching side-side-angle.
![[asy] pair A,B,C,D,F; A=(0,0);B=(5,4.04061);C=(6,0);D=(4,0);F=(5,0); draw(B--A--C--B--D); label("A",A,SW,red);label("E",F,S);label("B",B,NW,red);label("D",D,S,red);label("B",C,SE,blue);label("C",A,NW,blue);label("D",B,NE,blue);label("$\theta$",A,(4.5,1.5)); draw(anglemark(D,A,B,20));label("180",(A+B)/2,NW); draw(B--F,dashed);draw(rightanglemark(B,F,A));draw(D--B--C,dashed); add(pathticks(B--C,1,0.5,1,10));add(pathticks(B--D,1,0.5,1,10)); [/asy]](http://latex.artofproblemsolving.com/0/e/f/0efaa885cb080e2c72560aa4a3e7feefb71e621c.png)
Overlay the triangles
and
on each other as in the diagram above (where the red labels correspond to
and the blue labels correspond to
). Here we assume without loss of generality that
. Furthermore, let
be the angle
referenced in the problem; we need to find
.
Since the perimeter of
is
, we have
. Thus let
and
for some positive real number
. But the sides that correspond to
above are congruent, so we can drop a perpendicular from the topmost point to a point
, where the base of the isosceles triangle is bisected. Notice that the base of the isosceles triangle has length
, so in the diagram above,
.
Looking above at the right triangle containing
, we see that
. Hence
is our answer.
Solution 3
Start the same as solution 1. We get
where
is the length of
and
is the length of
. Let the common value of
and
be
. Then, the quadratic in
has solutions
and
. Therefore, by Vieta's,
. However, we know that the perimeter of
is
, so
, so
. Therefore,
See also
| 2003 AIME I (Problems • Answer Key • Resources) | ||
| Preceded by Problem 11 |
Followed by Problem 13 | |
| 1 • 2 • 3 • 4 • 5 • 6 • 7 • 8 • 9 • 10 • 11 • 12 • 13 • 14 • 15 | ||
| All AIME Problems and Solutions | ||
These problems are copyrighted © by the Mathematical Association of America, as part of the American Mathematics Competitions. Error creating thumbnail: Unable to save thumbnail to destination